You are on page 1of 60

Instructors Manual for

Industrial Organization
Theory and Applications

by Oz Shy

The MIT Press


Cambridge, Massachusetts
London, England

c 19952000 Oz Shy. All rights reserved.


Copyright 

The manual was typeset by the author using the LATEX 2 document preparation software
by Leslie Lamport (a special version of Donald Knuths TEX program) during the months
from September 1995 to November 1995 while I was visiting the Economics Department of
the University of Michigan. All the gures are also drawn in LATEX using a public domain
software called TEXcad developed by Georg Horn, which can be downloaded from various
mainframes.

Version 1.0 [Draft 20], (1995): Prepared for the rst printing of the rst edition
bkman21.tex rst draft to be posted on the Web (200/07/07)
(parallel to the 5th Printing of the book)
This version: BKMAN23.tex
2000/12/20 11:31

Contents
To the Instructor

Basic Concepts in Noncooperative Game Theory

Technology, Production Cost, and Demand

Perfect Competition

The Monopoly

Markets for Homogeneous Products

17

Markets for Dierentiated Products

25

Concentration, Mergers, and Entry Barriers

Research and Development

11

29

33

10 The Economics of Compatibility and Standards


11 Advertising

35

37

12 Quality, Durability, and Warranties

39

13 Pricing Tactics: Two-Part Tari and Peak-Load Pricing


14 Marketing Tactics: Bundling, Upgrading, and Dealerships
15 Management, Compensation, and Regulation
16 Price Dispersion and Search Theory
17 Miscellaneous Industries

53

51

49

43
45

To the Instructor
Before planning the course, I urge the instructor to read carefully the Preface of the book
that suggests dierent ways of organizing courses for dierent levels of students and also
provides a list of calculus-free topics.
The goals of this manual are:
To provide the instructor with my solutions for all the problems listed at the end of
each chapter;
to convey to the instructor my views of what the important concepts in each topic
are;
to suggest which topics to choose for dierent types of classes and levels of students.
Finally, please alert me to any errors or incorrect presentations that you detect in the
book and in this manual. (e-mail addresses are given below). Note that the errata les
(according to the printing sequence) are posted on the Web in PDF format. Before reporting
errors and typos, please view the errata les to see whether the error you found was already
identied and corrected.

Haifa, Israel, (December 20, 2000)


E-Mail: ozshy@econ.haifa.ac.il
Backup: (i) ozshy@post.tau.ac.il

(ii) ozshy@hotmail.com

Web Page: http://econ.haifa.ac.il/ozshy


Catalog Page: http://mitpress.mit.edu/book-home.tcl?isbn=0262691795

Chapter 2
Basic Concepts in Noncooperative Game Theory
An instructor of a short course should limit the discussion of game theory to the four most
important concepts in this chapter that are essential for the understanding most of the
analyses presented in this book:
1. The denition of a game (Denition 2.1): It is important that the student will understand that a game is not properly dened unless the list of players, the action set of
each player, and the payo functions are clearly stated. It is important that the student will understand the meaning of the term outcome as a list of the specic actions
chosen by each player (and not a list of payos as commonly assumed by students).
2. Nash equilibrium (Denition 2.4).
3. Welfare comparisons among outcomes (Denition 2.6).
4. Extensive form games, strategies (compare with actions), subgames, and the SPE
(Denition 2.10).
I urge the instructor to discuss the issues of existence, uniqueness, and multiple equilibria
in class. More precisely, it is important that the student will know that in order to prove
existence, it is sucient to nd only one NE outcome; however, to prove nonexistence, the
student must go over all outcomes and show that at least one player benets from unilateral
deviation.
I believe that the above can be covered in 2 lectures, or in three hours of instruction.
If you wish to devote more time to game theory, you can introduce the equilibrium in
dominant actions (Denition 2.3) before teaching the NE equilibrium concept. If you wish
to emphasis more game theory, I advise covering repeated games (section 2.3).
Answers to Exercises
1. (a) It is straightforward to conclude that

1

R (a ) =

(b)

WAR if a2 = WAR
WAR if a2 = PEACE


2

and R (a ) =

WAR if a1 = WAR
WAR if a2 = PEACE.

That is, WAR is each players best response to each action taken by the other
2 ) =
player (hence, WAR is a dominant action for each player). Now, (
a1 , a
(WAR, WAR) is a (unique) NE since this outcome is on the best-response function of each player.


RJ (aR ) =

if aR =
if aR =

and RR (aJ ) =

if aJ =
if aJ = .

Basic Concepts in Noncooperative Game Theory

(c)

There does not exist a NE for this game since there does not exist an outcome
the is on both best-response functions. That is, RJ () = , but RR () = , but
RJ () = , but RR () = , so RJ () = , and so on.


R (a ) =

B if a = L
T if a = R

and R (a ) =

L if a = T
R if a = B.

A NE does not exist for this game since R (L) = B, but R (B) = R, but
R (R) = T , so R (T ) = L, and so on.
2. (a) If (T, L) is a NE, then
(T, L) = a e = (B, L), and (T, L) = b d = (T, R).
(b) If (T, L) is an equilibrium in dominant actions, then T has to be a dominant
action for player , that is
(T, L) = a e = (B, L) and (T, R) = c g = (B, R);
and L is a dominant action for player , that is
(T, L) = b d = (T, R), and (B, L) = f h = (T, R).
Let us observe that the parameter restrictions given in part (a) are also included
in part (b) conrming Proposition 2.1 which states that an equilibrium in dominant actions is also a NE.
(c)

i. (T, L) Pareto dominates (T, R) if (a c and b > d) or (a > c and b d);


ii. (T, L) Pareto dominates (B, R) if (a g and b > h) or (a > g and b h);
iii. (T, L) Pareto dominates (B, L) if (a e and b > f ) or (a > e and b f ).

(d) Loosely speaking, the outcomes are Pareto noncomparable if one player prefers
(T, L) over (B, R), whereas the other player prefers (B, R) over the outcome
(T, L). Formally, either
(T, L) = a > g = (B, R) but (T, L) = b < h = (B, R),
or

(T, L) = a < g = (B, R) but (T, L) = b > h = (B, R).

3. (a) There are three Pareto optimal outcomes:


i. (n1 , n2 ) = (100, 100), where 1 (100, 100) = 2 (100, 100) = 100;
ii. (n1 , n2 ) = (100, 99), where 1 (100, 99) = 98 and 2 (100, 99) = 101;
iii. (n1 , n2 ) = (99, 100), where 1 (99, 100) = 101 and 2 (99, 100) = 98;
(b) There is no NE for this game.1 To prove this, we have to show that for every
outcome (n1 , n2 ), one of the players will benet from changing his or her declared
value. Let us look at the following outcomes:
1

A typo in the question (rst printing) leads to this undesirable result (see Basu, K. 1994. The Travelers
Dilemma: Paradoxes of Rationality in Game Theory. American Economic Review 84: 391395; for a
mechanism that generates (2, 2) as a unique NE outcome).

Basic Concepts in Noncooperative Game Theory

i. If (n1 , n2 ) = (100, 100), then player 1 can increase his or her payo to 1 =
101 by declaring n1 = 99.
ii. If (n1 , n2 ) = (99, 100), then player 2 can increase his or her payo from
2 = 98 to 2 = 99 by declaring n2 = 99. Hence, players always benet
from declaring a value of one dollar lower than the other player, and so on.
iii. If (n1 , n2 ) = (2, 2), then player 1 can increase his or her payo from 1 = 2
to 1 = 98 by declaring n1 = 100.
4. (a) There are several NE outcomes for this game. For example, (LG, SM, SM ) is a
NE outcome since
C (LG, SM, SM ) = = C (SM, SM, SM )
F (LG, SM, SM ) = = = F (LG, LG, SM )
G (LG, SM, SM ) = = = G (LG, SM, LG).
Hence, no rm nds it protable to unilaterally change the type of cars it produces.
(b) Again, there are several NE. The outcome (LG, SM, SM ) is also a NE for this
game, and the proof is identical to the one given in part (a).
5. (a) There are three subgames: the game itself, and two proper subgames labeled JL
(for Jacob left) and JR (for Jacob right). The three subgames are illustrated in
Solution-Figure 2.1.

R
=2
J = 1

JL

R
=0
J = 0

R
=2
J = 1

Rachel

Jacob

R
=0
J = 0

JR

R
=0
J
=0

R = 0
J = 0

R
=1
J = 2

R
=1
J = 2

Solution-Figure 2.1: Three subgames of the dynamic Battle of the Sexes


(b) One easy way to nd NE outcomes for an extensive form game is to construct
a normal-form representation.2 However, the normal-form representation is already given in Table 2.2. Also, equation (2.1) proves that the NE outcomes
involve the two players going together either to football or to the opera. Formally, it is easy to verify that the following three outcomes constitute NE:


sJ =

if sR =
if sR =

and

sR = ,

2
This subquestion may confuse students who are already thinking in terms of backward induction. Since
our analysis is based on SPE you may want to avoid assigning this subquestion (only). Also, an instructor
teaching from the rst printing is urged to change this subquestion to nding the NE for the entire game
only; hence, to postpone nding the NE for the subgames to the next subquestion, where the NE of the
subgames are used to nd the SPE.

Basic Concepts in Noncooperative Game Theory



J

s =

J

s =

if sR =
if sR =

and

sR = ,

if sR =
if sR =

and

sR = .
(2.1)

(c) Using backward induction, we rst construct Jacobs strategy (which constitutes
the Nash equilibria for the two proper subgames). Looking at the proper subgames in Solution-Figure 2.1, we conclude that

J

R (a ) =

if aR = (subgame JR)
if aR = (subgame JL).

Now, we look for Rachels strategy (given Jacobs best response). If Rachel plays
aR = , then her utility is given by R (, RJ ()) = 1. In contrast, if Rachel
plays aR = , then her utility is given by R (, RJ ()) = 2 > 1. Altogether, the
strategies

if aR =
R
R
J R
s = a = and R (a ) =
if aR =
constitute a unique SPE.
(d) No! Jacobs best response is to play whenever Rachel plays . Thus, Jacobs
announcement constitutes an incredible threat.3 Clearly, Rachel, who knows
Jacobs best-response function in the second stage, should ignore Jacobs announcement, since Jacob, himself, would ignore it if Rachel plays in the rst
stage.
6. (a) Jacobs expected payo is given by:
E J (, ) = 2 + (1 ) 0 + (1 ) 0 + (1 )(1 ) 1
= 2 + 3 .
Rachels expected payo is given by:
E R (, ) = 1 + (1 ) 0 + (1 ) 0 + (1 )(1 ) 2
= 2 + 3 2 2.
(b) The players best-response functions are given by

if < 1/3
[0, 1] if = 1/3
RJ () =

1
if > 1/3

and RR () =

if < 2/3
[0, 1] if = 2/3

1
if > 2/3.

The players best-response functions are drawn in Solution-Figure 2.2.


3

Instructors: Here is a good opportunity to discuss the concepts of credible and incredible threats.

Basic Concepts in Noncooperative Game Theory

RJ ()
1

RR ()
1

2
3


1
3

2
3

RJ ()

RR ()

1
3

Solution-Figure 2.2: Best-response functions for the Battle of the Sexes in mixed actions
(c) Solution-Figure 2.2 demonstrates that (, ) = (2/3, 1/3) is a NE in mixed actions. Also, note that the outcomes (0, 0) and (1, 1) are also NE outcomes in
mixed actions and are the same as the pure NE outcomes.
(d) Substituting (, ) = (2/3, 1/3) into the players payo functions (dened above)
yield
2
E J (2/3, 1/3) = E R (2/3, 1/3) = .
3
(e) The best-response functions in Solution-Figure 2.2 intersect three times, meaning
that in the mixed extension game, there are two equilibria: the two pure NE
outcomes and one NE in mixed actions. In contrast, Figure 2.3 has only one
intersection since in that game NE in pure strategies does not exist, and therefore
the best-response functions intersect only once, at the NE in mixed actions.

Chapter 3
Technology, Production Cost, and Demand
This chapter summarizes the basic microeconomic tools the students need to know prior
to taking this class. My advice for the instructor is not to spend time on this chapter
but simply assign this chapter (with or without the exercises) as reading in the rst class.
However, my advice is to return (or refer) to this chapter whenever denitions are needed.
For example, when you rst encounter a discussion of returns to scale, I urge you to make
a formal denition and refer the students to Denition 3.2. Similarly, when addressing
elasticity issues, students should be referred to Denition 3.3.
Answers to Exercises
1. (a) Let > 1. Then, by Denition 3.2, the technology exhibits IRS if
(l) (k) = + l k > l k ,
which holds when + > 1. Using the same procedure and Denition 3.2, this
technology exhibit CRS if + = 1, and DRS if + < 1.
(b) In this technology, the factors are supporting since
M PL (l, k)

Q
= l1 k ,
l

hence,

M PL (l, k)
= l1 k 1 ,
k
which is greater than zero under the assumption that , > 0.

2. (a) Let > 1. Then, by Denition 3.2, the technology exhibits IRS if
(l) + (k) = (l + k ) > (l + k ),
which holds if > 1. Similarly, the technology exhibits DRS if < 1, and CRS
if = 1.
(b) M PL (l, k) = l1 . Hence, M PL (l, k)/k = 0. Therefore, the factors are
neither substitutes nor complements.
3. Let > 1. This (quasi-linear) technology exhibits DRS since

l + k < l + k = (l + k).
4. (a)
AC(Q) =

F
T C(Q)
T C(Q)
=
+ c, and M C(Q) =
= c.
Q
Q
Q

These functions are drawn in Figure 4.2 in the book.

Technology, Production Cost, and Demand


(b) Clearly, AC(Q) declines with Q. Hence, AC(Q) is minimized when Q = +.
(c) Declining average cost function reects an increasing returns to scale technology.
5. (a) Using Denition 3.3,
p (Q)

99 Q
Q(p) p
p
.
= (1) =
Q
Q
Q Q

Hence, p (Q) = 2 when Q = 33.


(b) From the above, p (Q) = 1 when Q = 99/2 = 49.5.
(c) The inverse demand function is given by p(Q) = 99 Q. Therefore, T R(Q) =
p(Q)Q = (99 Q)Q. Hence, M R(Q) dT R(Q)/dQ = 99 2Q, which is a
special case of Figure 3.3.
(d) M R(Q) = 0 when Q = 49.5. That is, the marginal revenue is zero at the output
level where the demand elasticity is 1 (unit elasticity).
(e) Using Figure 3.5,
CS(33) =

(99 33)66
= 2178,
2

and CS(66) =

(99 66)33
= 544.5
2

6. (a)
p
=

A
. Hence,
Q

p = A  Q  .

(b)
p =

p
Q(p) p
A(+)p

= +.
= A(+)p
1 =
Q Q
Q
Ap

Thus, the elasticity is constant in the sense that it does not vary with the quantity
consumed.
(c) The demand is elastic if p < 1, hence if + > 1. The demand is inelastic if
1 < p 0, hence if 0 + < 1.
(d) By Proposition 3.3, M R = p[1 + 1/(+)]. Hence,
p
=1
MR
which is independent of Q.

1+

1
+
=
+
+1

Chapter 4
Perfect Competition
A perfectly competitive market is characterized by nonstrategic rms, where rms take the
market price as given and decide how much to produce (and, whether to enter, if free entry
is allowed). Most students probably had some discussion of perfectly competitive markets
in their intermediate microeconomics class. However, given the importance of this market
structure, I urge the instructor to devote some time in order to make sure that the students
understand what price-taking behavior means.
It is now the right time to emphasize that, in economics, the term competitive refers to
price taking behavior of agents. For example, we generally assume that our consumers are
competitive, which means that they do not bargain over prices and take all prices and their
income as given. In a competitive market structure, we make a similar assumption about
the rms.
You may want to emphasize and discuss the following points:
1. The assumption of price taking behavior has nothing to do with the number of rms
in the industry. For example, one could solve for a competitive equilibrium even
in the presence of one rm (see an exercise at the end of this chapter). Note that
this confusion often arises since certain market structures yield market allocations
similar to the competitive allocation when the number of rms increases (see for
example subsection 6.1.2 which shows that the Cournot allocation may converge to
the competitive allocation when the number of rms increases to innity).
2. A major reason for studying and using alternative (noncompetitive) market structures
stems from the fact that the competitive market structure very often fails to explain
why concentrated industries are observed.
3. Another major reason for studying alternative market structures stems from the
nonexistence of a competitive equilibrium when rms technologies exhibit IRS.
Finally, note that this chapter does not solve for a competitive equilibrium under decreasing
returns to scale technologies for two reasons: (i) other market structures analyzed in this
book are also developed mainly for CRS (unit cost) technologies, and (ii) most students are
familiar with the DRS from their intermediate microeconomics class. However, the exercise
at the end of this chapter deals with a DRS technology.
Answers to Exercises
1. Firm 1 takes p as given and chooses q1 to


max 1 = pq1 wL1 = p L1 wL1 .


q1

10

Perfect Competition
The rst and second order conditions are given by
0=
Hence, q1 =

d1
p
= w,
dL1
2 L1

d2 1
p
= 3/2 < 0.
2
d(L1 )
4L
1

L1 = p/(2w).

2. Given that there is only one rm, the supply equals demand equilibrium condition
yields that 120 pe = pe /2. Therefore, pe = 80; hence, q1e = Qe = pe /2 = 40.
3. From the production function, we can nd the equilibrium employment level to be
Le1 = (q1e )2 = 1600. Hence,
1 = pe q1e wLe1 = 80 40 1600 = 1600.
4. Given that the rms have the same technologies, they have the same supply functions.
Therefore, the supply equals demand condition becomes 120pe = pe /2+pe /2, which
yields pe = 60, Qe = 120 60 = 60, hence, q1e = q2e = 30.
5. Clearly, the competitive price is lower and the aggregate production is higher when
the industry consists of two rms.
p

p = 2q1

60

p = 2q2

30

p
120

q1

30

p = q1 + q2

q2

60

p = 120 Q
120

Q = q1 + q2

6.
Solution-Figure 4.1: Competitive equilibrium with two rms

Chapter 5
The Monopoly
Most students encounter the monopoly problem in their intermediate microeconomics class.
However, the instructor would probably want to make sure that all students fully understand
the monopolys choice problem and the eect of price elasticity on the monopolys price, as
well as the arguments against monopoly (section 5.2). I also urge the instructor not to skip
discussing discriminating monopoly (section 5.3).
The remaining sections, the cartel (section 5.4), and the durable goods monopolies
(section 5.5) are more optional depending on the instructors tastes and students ability.
Answers to Exercises
1. (a)
p

p
dQ p
= a+p
1
= +.
ap
dp Q

Hence, the exponential demand function has a constant price elasticity. By


Proposition 3.3,




+1
1
M R(Q) = p 1 +
=p
.
+
+
(b) Equating marginal revenue to marginal cost yields1
MR = p

+1
+

= c = M C;

hence, pM =

+c
.
+1

(c) As + increases, the demand becomes more elastic, hence, the monopoly price
must fall. Formally,
dpM
c(+ 1) +c
< 0.
=
d+
(+ 1)2
(d) The rst edition (rst printing) contains a typo. The question asks what happens
to the monopolys price when + +1. Clearly, pM +. The reason is, that
when + = 1, the (entire) demand has a unit elasticity, implying that revenue
does not vary with price (or quantity produced). Hence, given that the revenue
is constant (in fact T R = a when + = 1), then the prot maximization problem is
reduced to cost minimization which yields that the monopoly would attempt
to produce as little as possible (but still a strictly positive amount).
1

The instructor may want to emphasize to the students that in the case of constant-elasticity demand
functions (exponential demand functions) it is easier to solve for the monopolys price rst (using Proposition 3.3) and then solve for the quantity produced by substituting the price into the demand function. This
procedure becomes very handy when solving monopolistic competition equilibria analyzed in section 7.2.

12

The Monopoly
(e) Inverting the demand function yields p(Q) = a1/
Q1/
. Thus,
1

T R(Q) p(Q)Q = a  Q1  .
Hence,

M R(Q)

1
1
1
dT R(Q)
= a  Q  1
.
dQ
+

(f) Equating marginal revenue to marginal cost yields


1


1

a Q

1
1
+

= c,

yielding that the monopolys prot maximizing output is


M

+1
=a
+c

Note that the same result is achieved by substituting pM (calculated before) into
the demand function.
2. (a) Solution-Figure 5.1 illustrates an aggregate demand composed of the two groups
of consumers, where each group shares a common valuation for the product.
p
VH

VL
nH

nH + nL

Solution-Figure 5.1: Aggregate demand composed of two consumer groups


(b) The monopoly has two options2 : setting a high price, p = V H , or a low price, p =
V L . Solution-Figure 5.1 reveals that the prot levels (revenue since production
is costless3 ) are given by
|p=V H = nH V H ,

and |p=V L = (nH + nL )V L .

Comparing the two prot levels yields the monopolys prot maximizing price.
Hence,

V H if V H > (nH + nL )V L /nH
M
p =
V L otherwise.
2

Instructors are urged to assign or discuss this exercise, since it provides a good opportunity to introduce
the student to a discrete (logic based) analysis which is used later in a wide variety of topics (see for example
the section on tying [section 14.1]).
3
The rst printing of the rst edition neglects to assume that production of G-Jeans is costless.

The Monopoly

13

Thus, the monopoly sets a high price if either there are many high valuation
consumers (nH is large) and/or these consumers are willing to pay a very high
price (V H is high).
3. (a) In market 1, p1 = 2q1 .
M R1 (q1 ) = c = 1 yields
In market 2, p2 = 4q2 .
M R2 (q2 ) = c = 1 yields

Hence, by Proposition 3.2, M R1 (q1 ) = 22q1 . Equating


q1 = 0.5. Hence, p1 = 1.5.
Hence, by Proposition 3.2, M R2 (q2 ) = 42q2 . Equating
q2 = 1.5. Hence, p2 = 2.5.

(b) 1 = (p1 c)q1 = (0.5)2 = 0.25, and 2 = (p2 c)q2 = (1.5)2 = 2.25. Summing
up, the monopolys prot under price discrimination is = 2.5.
(c) There are two cases to be considered: (i) The monopoly sets a uniform price
p 2 thereby selling only in market 2, or (ii) setting p < 2, thereby selling a
strictly positive amount in both markets. Let us consider these two cases:
i. If p 2, then q1 = 0. Therefore, in this case the monopoly will set q2
maximize its prot in market 2 only. By subquestion 3a above, = 2 =
2.25.
ii. Here, if p < 2, q1 > 0 and q2 > 0. Therefore, aggregate demand is given
by Q(p) = q1 + q2 = 2 p + 4 p = 6 2p, or p(Q) = 3 0.5Q. By
Proposition 3.2, M R(Q) = 3 Q. Equating M R(Q) = c = 1 yields Q = 2,
hence, p = 2. Hence, in this case = (p c)2 = 2 < 2.25.
Altogether, the monopoly will set a uniform price of p = 2.5 and will sell Q = 1.5
units in market 2 only.4
4. (a)
(q1 , q2 ) = (100 q1 /2)q1 + (100 q2 )q2 (q1 + q2 )2 .
(b) The two rst order conditions are given by

q1

0=
q2

0=

= 100 q1 2(q1 + q2 )
= 100 2q2 2(q1 + q2 ).

Solving for q1M and q2M yields that q1M = 25 and q2M = 12.5.
(c) Substituting the prot maximizing sales into the market demand functions yield
M
pM
1 = p2 = 87.5. Hence,
(q1M , q2M ) = 87.5 12.5 + 87.5 25 (12.5 + 25)2 = 1875.
4
Note that consumers in market 1 are better o under price discrimination than without it, since under
no discrimination no output is purchased in market 1. Given that the price in market 2 is the same under
price discrimination and without it, we can conclude that in this example, price discrimination is Pareto
superior to nonprice discrimination, since both consumer surplus and the monopoly prot are higher under
price discrimination.

14

The Monopoly
(d) Now, that each plant sells only in one market, the two rst order conditions
become

q1

0=
q2
0=

= 100 q1 2q1
= 100 2q2 2q2 .

yielding q1M = 100/3 and q2M = 25.

M
(e) pM
1 = 100 100/6 = 250/3 and p2 = 100 25 = 75. Hence,

250 100

= 1 (q1 ) + 2 (q2 ) =
3
3

100
3

+ 75 25 252 = 2917.

(f) This decomposition increases the monopolys prot since the technology exhibits
DRS.
5. By section 5.3, the discriminating monopoly will set quantities to satisfy M R1 (q1M ) =
M R2 (q2M ). Hence, by Proposition 3.3
M R1 (q1 ) =

pM
1

1
1+
2

pM
2

1
1+
4

= M R2 (q2 ).

M
Thus, pM
1 = 1.5p2 .

6. (a) We solve for the monopolys prot maximizing prices starting from the second
period. The second period outcome may depend on two cases:
First-period consumer does not buy in period 1: Clearly, in this case, the
second period prot maximizing price is p2 = 50, yielding a prot level of
2 = 3 50 = 150.
First-period consumer buys in period 1: In this case the second period prot
maximizing price is again p2 = 50, yielding a prot level of 2 = 250 = 100.
Altogether, the second-period price is independent of the action of the rstperiod buyer in the rst period. Therefore, the maximum price the monopoly
can charge the rst-period buyer in the rst period is p1 = 150.
(b) The second period outcome may depend on two cases:
First-period consumer does not buy in period 1: In this case, the monopoly
has two choices: (i) charging p2 = 20, and sell to all three consumers, thereby
earning a second period prot of 2 = 3 20 = 60; or, (ii) charging p2 = 50,
and selling only to the second period consumers, thereby earning a secondperiod prot of 2 = 2 50 = 100.
First-period consumer buys in period 1: In this case the second period prot
maximizing price is again p2 = 50, yielding a prot level of 2 = 250 = 100.
Altogether, the second period price is independent of the actions of the rstperiod buyer. Now, in order to attract the rst-period buyer to purchase in
period 1, the monopoly should set p1 = 40, thereby extracting all surplus from
all consumers.

The Monopoly

15
p

pM (c + t)

a
pM (c + t)
pM (c)

c+t

c+t

pM (c)

D
Q

MR

M R(Q)
Solution-Figure 5.2: How the monopoly price varies with a specic tax
7. The two cases are illustrated in Solution-Figure 5.2.
(a) Equating marginal revenue to the tax inclusive unit cost yields a 2Q = c + t,
or QM = (a c t)/2. Hence, pM = (a + c + t)/2. Now, checking the eect of a
tax rate change on the monopolys price yields that dpM /dt = 1/2 < 1. Hence,
as illustrated in Solution-Figure 5.2 (left), an increase in t raises the monopoly
price by less than t.
(b) Using Proposition 3.3,


M R = pM 1 +

1
2

= c + t,

yielding pM = 2(c + t).

Now, checking the eect of a tax rate change on the monopolys price yields that
dpM /dt = 2 > 1. Hence, as illustrated in Solution-Figure 5.2 (right), an increase
in t raises the monopoly price by more than t.

Chapter 6
Markets for Homogeneous Products
The Cournot, Bertrand, and sequential moves market structures are essential for a further
study of industrial organization.
The important issues to discuss and emphasize are:
1. Market structures are assumed by the researcher rather than solved for. Specifying
a market structure is the same as specifying the rules of the game before predictions
(e.g., equilibrium outcomes) are sought.
2. Cournot and Bertrand market structures select Nash equilibrium outcomes, where
under the Cournot game rms actions are the quantity produced, and in the Bertrand
game rms actions are prices.
3. Free entry versus a xed number of rms.
Self-enforcing collusion (section 6.5) is suited for more advanced students. Instructors
may want to go over repeated games (section 2.3) before covering self-enforcing collusion,
however, this topic is written in a way that enables the instructor to teach self-enforcing
collusion even without formally teaching repeated games. Instructors who choose to teach
this topic may want to emphasize that a trigger strategy is a function of the entire history of what each player has played in earlier periods, including the players own earlier
actions, and therefore provides self-discipline which restricts the gains from deviation from
the cooperative actions.
Finally, subsection 6.6.2 (preferential trade agreements) provides a good example to the
law of (no) second best, by showing that a partial removal of trade barriers need not be
welfare improving.
Answers to Exercises
1. (a) By Lemma 4.1, both rms produce a nite amount of output only if p c1 and
p c2 . Hence, since c2 > c1 , if a competitive equilibrium exists then it must
be that pe c1 . However, if pe < c1 then both rms produce zero output, but
at these prices demand exceeds zero (since Q(c1 ) = c1 > 0). Hence, if a
competitive equilibrium exists, then pe = c1 . Therefore, q1e = Qe = c1 and
q2e = 0.
(b) Each rm i takes the output of its opponent, qj , as given and solves
max i = ( qi qj )qi ci qi , i, j = 1, 2, i = j,
qi

yielding rst order conditions given by


0=

1
= 2q1 q2 c1 ,
q1

and 0 =

2
= q1 2q2 c2 .
q2

18

Markets for Homogeneous Products


Solving the two equations for the two variables, q1 and q2 yields
q1c =

2 c1 c2
2c2 + c1
2c1 + c2
, and Q =
.
, q2 =
3
3
3

Then, pc = q1c q2c = ( + c1 + c2 )/3.


(c) From the rst order conditions of the previous subquestion we can immediately
solve for rm 2s best-best response function. Hence,
R2 (q1 ) =

c2 q1
.
2
2

In a sequential-moves equilibrium, rm 1 (the leader) takes rm 2s best-response


function as given and chooses q1 to

max 1 = q1
q1

c2 q1

2
2

q1 c1 q1 ,

yielding the rst order condition


0=

c2
1
= 2q1
+ q1 c1 .
q1
2

Therefore,
q1s =

+ c2 2c1 s 3c2 + 2c1


3 c2 2c1
, q2 =
, and Qs =
.
2
4
4

Hence, ps = ( + c2 + 2c1 )/4.


(d) When rm 2 is the leader, by symmetry (replacing c1 by c2 and vice versa in the
previous subquestion) we can conclude that
q2s =

+ c1 2c2 s 3c1 + 2c2


3 c1 2c2
, q1 =
, and Qs =
.
2
4
4

Hence, ps = ( + c1 + 2c2 )/4.


Comparing the two sequential-moves equilibria we see that (i) aggregate output
decreases when the less ecient rm (rm 2) moves before rm 1; (ii) price is
higher when rm 2 moves rst; (iii) the market share of the less ecient rm
increases when it moves before rm 1; (iv) the production of the more-ecient
rm increases when it moves rst; (v) the production of the less-ecient rm
decreases when it moves rst; and (vi) the market share of the more-ecient
rm increases when it moves rst.
(e) In a Bertrand equilibrium, the more ecient rm (rm 1) completely undercuts
the price set by rm 2. Assuming that money is continuous, rm 1 sets its price to
equal the unit cost of rm 2, thereby ensuring that rm 2 will not nd it protable
to produce any amount. Hence, we can approximate the Bertrand equilibrium
by pb1 = c2 + and pb2 = c2 . Thus, q1b = Qb = c2 + and q2b = 0. Notice that
when the unit costs are not equal (the present case), the Bertrand equilibrium
price exceeds the competitive equilibrium price. Also, instructors should point
out to the students that this is not really a Nash-Bertrand equilibrium since the
prot of rm 1 increases as + decreases.

Markets for Homogeneous Products

19

2. (a) We rst focus on the problem faced by rm 1. In a Cournot market structure,


rm 1 takes q2 , q3 , . . . , qN as given and chooses q1 that solves
max 1 = (100 q1 q2 qN )q1 F (q1 )2 .
q1

The rst order condition is given by


0=

1
= 100 2q1 q2 qN 2q1 .
q1

Therefore, the best-response function of rm 1 is given by


q1 = R1 (q2 , . . . , qN ) =

100

j=2 qj

Given that all rms have identical technologies (identical cost functions), we can
attempt to search for a Cournot equilibrium where all rms produce the same
c . In this case,
output levels. That is, q c q1c = q2c = = qN
q c = 25

(N 1)q c
100
100N
, or q =
, hence, Q =
.
4
N +3
N +3

Therefore,
pc = 100

300
100N
20, 000
=
, and i = pc q c F (q c )2 =
F.
N +3
N +3
(N + 3)2

(b) Under free entry, rms enter as long as they make above normal prots. Entry
stops when a further entry generates a loss to rms. Hence, if we approximate
the number of rms, N , by a real number, under free entry all existing rms
make zero prot. Thus,
20, 000
F, or N =
0 = i =
(N + 3)2

20, 000
3.
F

Thus, the (endogenously determined) number of rms in this industry declines


with the xed cost parameter F .
3. In the third period, rm 3 takes q1 and q2 as given and chooses q3 to maximize its
prot. The solution to this problem yields rm 3s best-response function R3 (q1 , q2 )
which is given in (6.8). Hence, using (6.8) we have it that
R3 (q1 , q2 ) =

120 c q1 + q2

.
2
2

In the second period, rm 2 takes q1 and R3 (


q1 , q2 ) as given and chooses q2 that solves
max 2 = [120 q1 q2 R3 (
q1 , q2 )]q2 cq2 .
q2

20

Markets for Homogeneous Products


The rst-order condition is given by 0 = d2 /dq2 = 60 q1 /2 q2 c/2. Therefore,
the best-response function of rm 2 is given by
R2 (q1 ) =

120 q1 c
.
2

In the rst period, rm 1 takes the best-response functions of rms 2 and 3 as given
and chooses q1 that solves
max 1 = {120 q1 R2 (q1 ) R3 [q1 , R2 (q1 )]}q1 cq1 = {30 +
q1

3c q1
}q1 cq1 .
4
4

The rst-order condition is given by 0 = d1 /dq1 = 30c/4q1 /2. Therefore, solving


for q1s , and then substituting into R2 (q1s ), and then into R3 [q1s , R2 (q1s )] yields,
q1s =

120 c
120 c s 120 c
, q2 =
, and q3s =
.
2
4
8

Hence, Qs = q1s + q2s + q3s = 7(120 c)/8 and ps = (120 + 7c)/8.


4. (a) Under zero production cost, all Bertrand equilibrium outcomes yield zero prots
to both rms. Thus the Bertrand outcome is pb1 = 0 and pb2 = 0.
(b) Well construct an equilibrium in trigger strategies where rms threat point is
pb1 = 0 and pb2 = 0, respectively. Formally, let rm is price strategy, i = 1, 2, be
given by


pi ( ) =

10 as long as p1 (t) = p2 (t) = 10 for all t = 1, . . . 1


0 otherwise.

Now, if no rm deviates in any period, the present value of the sum of discounted
prot to each rm i is i = 5N/(1 ). If rm 1 deviates in period , then
1 = (10 +)N + 0 10N (since eective period + 1 all rms revert to pi = 0).
Hence, deviation is not protable to either rm if 5N/(1 ) > 10N , or > 1/2.
(c) All Bertrand equilibria are of the form pb1 = 4 + 4 and pb2 = 4.
(d) We can reconstruct rms trigger strategies where we assume that the punishment
point is now pb1 = 4 and pb2 = 4. Since now rms have dierent prot functions,
we need to check protability from deviation by each rm separately.
Firm 1: In each period , if no rm deviates, the sum of discounted prots is
1 = 5N/(1 ). If rm 1 deviates in period , then its sum of discounted
prot is 1 = 10N + 4N/(1 ). Comparing the two prot levels shows
that deviation is not protable for rm 1 if > 5/6.
Firm 2: If no rm deviates, the sum of discounted prot is 2 = (N/2)(10
4)/(1). If it deviates, then the sum of discounted prot is 2 = (104)N +
0. Comparing the two prot levels reveals that deviation is not protable if
> 1/2.
(e) Collusion is less likely to be sustained when one rm has a cost advantage over
the other because deviation by rm 1 (low cost) will leave it with a positive
stream of prot compared with a zero prot generated by deviation when the
two rms have an identical cost structure.

Markets for Homogeneous Products

21

5. (a) Let pA
k denote the (tari inclusive) price consumers in country A pay for a unit
of import from country k, k = B, C. Therefore, under a uniform specic tari
A
of $10 per unit of import, pA
B = 70 and pC = 50. Clearly consumers buy from
country C, and the government earns a revenue of $10 per unit. A FTA with
A
A
country B reduces pA
B to pB = 60 > 50 = pC . Hence, As consumers still purchase
from country C implying that this FTA is ineective and therefore does not alter
As welfare.
A
A
(b) Now, a FTA with country B reduces pA
B to pB = 60 < 60.01 = pC . Hence, As
consumers switch to buying from country B and no tari revenues are collected.
A
Note that in this case the consumer price falls from pA
C = 60.01 to pB = 60 (a
reduction of 1 cent per unit of import). Hence, the FTA hardly changes consumer
surplus. However, in this case the government faces a large reduction in tari
revenues due to the elimination of a $10 tari per unit of import. Altogether,
with the exception of highly elastic demand, country A loses from the FTA.

6. (a) Clearly pL is a dominant action for each rm in the static one-shot game. Hence,
by Proposition 2.2, (pL , pL ) is a unique NE for the one-shot game. Therefore, if
a SPE exists, it must yield that both rms charge pL .
We can derive the SPE directly by formulating the extensive form game which
is illustrated in Solution-Figure 6.1 (consider the rst two stages only).
Firm 1

(I.:)
pL
pH
(II.:)
pL

Firm 2
pH pL

1 = 120
2 = 0

1 = 70
2 = 70
(III.:)

pL

pH

1 = 0
2 = 120

1 = 100
2 = 100

Firm 1
1 = 70
2 = 70

Solution-Figure 6.1: Sequential price game: Meet the competition clause


From the gure, it is straightforward to conclude that the following prices constitute a SPE:


p2 =

pL if p1 = pH
pL if p1 = pL

and

p1 = pL .

(b) We now add a third stage to the game, where rm 1 commits to reduce its price to
rm 2s price whenever rm 2 charges pL . This third stage is (partly) illustrated
at the bottom of Figure 6.1. In this case, the SPE if given by


p2 =

pH
pL

if p1 = pH
if p1 = pL

and p1 = pH ,

22

Markets for Homogeneous Products


implying that the rms charge the industrys prot maximizing price and earn a
prot of 100 each.
7. Note that the answer for exercise 3 provides a solution for N = 3 rms, which intuitively can be generalized to the N > 3 case. In addition, in what follows, I provide
a formal solution for this problem.
(a) Let qis denote the sequential-moves equilibrium output level produced by rm i,

N 1 s
s
s
i = 1, . . . , N . Dene Qs N
i=1 qi and QN =
i=1 qi , for i = 1, . . . , N where
Qs1 0 for i = 1.
Since there is no xed cost, there is no limit on the number of rms that will
enter if free entry is allowed. We therefore look at how the N th rm decides on
how much to produce. Firm N solves
max N = (a QsN qN c)qN ,
qN

s = (a Qs
yielding qN
N c)/2.
Before we proceed to analyzing rm N 1, let us dene aN a QsN . Hence,
it can be veried that
s
aN 1 = aN + qN
1

for allN.

Firm N 1 knows what rm N will be producing and solves

qN 1

[aN c]qN 1
2

aN c
c qN 1
2
[aN 1 qN 1 c]qN 1
=
.
2

s
(a Qs(N 1) qN
1 )

max N 1 =

s
Hence, qN
1 = (aN 1 c)/2.
s
s
Also, since aN = aN 1 qN
1 and aN = aN 1 qN 1 = (aN 1 + c)/2,
s
= (aN c)/2 = (aN 1 c)/4.
qN
s
We now analyze rm N 2. Recall that aN 2 qN
2 = aN 1 . Also, recall that
s
s
qN + qN 1 = (1/2 + 1/4)(aN 1 c). Therefore, rm 2 solves

max N 2 =
qN 2

3(aN 2 qN 2 c)
c qN 2
4
[aN 2 qN 2 c]qN 2
.
4
(aN 2 qN 2 )

s
s
s
The solution is qN
2 = (aN 2 c)/2. Hence, qN 1 = (aN 2 c)/4 and qN =
(aN 2 c)/8. Therefore,
s
qN
=

aN c
,
2

for all N.

In particular, for N = 1, a1 = a Qs1 = a 0 = a. Thus, q1s = (a c)/2.

Markets for Homogeneous Products


Finally, since
s
qN
=

we have it that
s
qN

 k

1
2

23

s
qN
k ,

 N 1
1

for all 0 < k < N,

q1s =

1
2N 1

ac
ac
= N .
2
2

Altogether, the N -rm sequential moves equilibrium output levels are


qis =

ac
,
2i

for all i = 1, . . . , N.

(b) Aggregate output is given by


s

Q =


i=1

N qis

1 1
1
= (a c)
+ + + N
2 4
2

Using the same arguments as in the proof of Lemma 9.2 (see mathematical
appendix, section 9.9), we can show that1


1
Q = (a c) 1 N
2
s

(c) As the number of rms increases, Qs a c (competitive output level). Hence,


ps c (the competitive price level).

The rst printing of the rst edition contains a typo in the specication of Qs in the question.

Chapter 7
Markets for Dierentiated Products
The rst section extends the market structures dened in Chapter 6 to markets with dierentiated products. Using the two-brand environment enables us to analyze the dierence
in market outcomes generated by the Cournot and the Bertrand market structures, and to
dene and contrast the concepts of strategically substitutes and strategically complements
best-response functions.
Section 7.2 (somewhat more advanced) introduces the student to the endogenous determination of the variety of brands, and its application to the theory of international trade.
Location models (section 7.3), in particular the Hotelling model (subsection 7.3.1), are
important for a further study of industrial organization. The linear city model is used several
times in the manuscript, for example, to distinguish between vertically and horizontally
dierentiated products (see section 12.2). Instructors should cover at least the Hotelling
linear city model which students always nd very intuitive and therefore very appealing. The
existence proof (Appendix, section 7.5) can be skipped. The instructor can demonstrate the
parameter restrictions needed to ensure existence by imposing symmetric locations (a = b)
into the conditions of Proposition 7.6.
Answers to Exercises
1. (a) The best-response functions are drawn in Figure 7.1 and are derived as follows:
3
max 1 R1 (2 ) = 2 + 2 ,
1
2
1
max 2 R2 (1 ) = 1 + 1 .
2
2

4 2
or, 2 = + R1 (2 );
3 3

R1 (2 )
8

R2 (1 )

1
0
43

14

Solution-Figure 7.1: Advertising best-response functions

26

Markets for Dierentiated Products


(b) The two best-response functions are upward sloping, hence strategically complements.
(c) Solving for a NE yields 1N = 14 and 2N = 8. The rms prot levels in a NE
are: 1 = 142 and 2 = 82 .
2. (a) Given a low reservation price (a low B), well attempt to nd a monopoly equilibrium where not all the market is served. Figure 7.2 illustrates the location of the
two indierent consumers, one on each side of the rm. The indierent consumer

1
2

2a

1
2

1
2

+a

Solution-Figure 7.2: Single-rm location model


(on each side) is determined by the reservation utility, that is, B a p = 0, or,
a = B p. Hence, the monopoly chooses p that solves
max = p2a = 2p(B p).
p

The solution is given by


pM =

B
,
2

a=

B
,
2

and M = 2pa =

B2
.
2

Now, it is easy to verify that 0 < B < 1 implies that 0 < a < 1/2, hence, not all
the market is served.
(b) When substituting B > 1 into the solution for a found in the previous subquestion, we get that the indierent consumers lie outside the city. This implies
that for B > 1, the monopoly can increase the price and still having the entire
street purchase the product. Thus, the monopoly will pick the highest possible
price subject to having the consumers living at the edges of town purchase the
product. Formally, set p to satisfy B 1/2 p = 0. Hence,
1
a= ,
2

1
pM = B ,
2

1
and M = B .
2

3. (a) The indierent consumer, denoted by x


, must satisfy
) R + p2 .
x
1 + p1 = (1 x
Hence,
x
=

R + p2 p 1
.
1+R

= R/(1 + R). Hence,


(b) Substituting p1 = p2 yields x
lim x
= lim

R
= 1.
1+R

Markets for Dierentiated Products

27

Therefore, all the consumers will eat in restaurant 1 (i.e., x


= 1) only if the transportation cost for traveling to the east is innite. Otherwise, some consumers
will always eat at restaurant 2.
4. From (7.24), we can dene the location of the indierent consumer, x
, by the implicit
function:
F (pA , pB ) pA pB + (
x a)2 (
x L + b)2 = 0.
subject to x
satisfying F (pA , pB ) = 0.
Firm A chooses pA that solves maxpA A = pA x
The rst-order condition is given by


dA
p
0=
=x
+ pA
=x
+ pA FA
dpA
pA
x

=x

pA
.
2 (L a b)

), yielding a rst-order
Similarly, rm B chooses pB that solves maxpB B = pB (L x
condition given by
x

dB
=Lx
pB
=x
+ pB
0=
dpB
pB

 F 
pB
F
x

=Lx

pB
.
2 (L a b)

(a) Under a = b, we search for a symmetric solution. Substituting a = b and x


= L/2
into rm As rst-order condition (or Bs rst-order condition) yields that
pA = pB = L(L 2a).
Note that pA = pB 0 when a 1/2, reecting the fact that prices drop to
zero when the two brands become homogeneous.
(b) We now investigate the total eect of varying the location of rm A (varying a)
on the prot of rm A.1
Using the two rst-order conditions, we have it that pA = 2 (L a b)
x and
pB = 2 (L a b)(L x
). Substituting into the function F (, ) = 0 yields
x L + b)2 = 0.
F (, ) = 2 (L a b)(2
x L) + (
x a)2 (
Using the implicit function theorem, we have it that


x

1
(2
x L) 2(
x a)
=
=x=1/2 .
a
2(L a b) + 2(
x a) 2(
x L + b)
4
a=b

=
Now, using As rst-order condition, we can express As prot by A = pA x
2
2 (L a b)(
x) . Therefore,
dA
<0
da

whenever

1
(
x)2 + (L a b)2
x < 0,
4

which always hold since L 2a < L. Hence, rm A increases its prot by moving
toward point 0. Similarly, rm B increases its prot by moving toward point L.
1
This problem may be too dicult for most undergraduate students since it involves deviating from the
symmetric solution.

Chapter 8
Concentration, Mergers, and Entry Barriers
This chapter contains four major topics: How to measure concentration; merger and merger
guidelines; entry barriers; and entry deterrence. I urge the instructor not to skip the
discussion of concentration measures (section 8.1) and the appendix on merger regulations
and guidelines (section 8.6).
Regarding all other topics, the instructor will nd a wide variety of models to choose
from. Most of the topics are logical and require minimum calculations and very little calculus
(if any).1 Finally, note that you can combine the topic of double monopoly markup, analyzed
under the topic of dealership (subsection 14.3.1), with the analysis of vertical integration
given in subsection 8.2.2.
Answers to Exercises
1. (a) I4 = 20 + 20 + 20 + 10 = 70.
(b) IHH = 4 102 + 3 202 = 1, 600.
(c)

i. After the merger of rms 1 and 2, IHH = 202 + 102 + 102 + 3 202 = 1, 800.
ii. IHH = 1, 800 1, 600 = 200.
iii. The merger may be challenged by the FTC or the Justice Department since
the postmerger IHH exceeds 1, 000 and IHH = 200 > 100.

2. (a) The solution for a Cournot equilibrium with N rms is given in (6.9) and (6.10).
Substituting c = 0 (zero production cost) and N = 3 yields qic = 25 and ic = 625,
i = 1, 2, 3.
(b) Substituting N = 2 into (6.10) yields 1 = 4 = 1111.111 . . ..
(c) Since rms 2 and 3 split the prot from the merged rm, 2 = 3 = 4 /2 =
555 < 625. Hence, rms 2 and 3 lose from this merger whereas rm 1 gains.
(d) A merger into a monopoly cannot reduce aggregate prot, simply because a
monopoly can always mimic the same production pattern of any oligopoly. To
see this, substituting N = 1 into (6.10) yields a monopoly prot of M = 2500.
Hence, i = 2500/3 = 833 13 > 625 > 555. Thus, the prot per rm is the highest
under the monopoly market structure (cartel).
(e) The dierence between the two mergers is as follows: When two out three rms
merge, competition is reduced to a duopoly; however, the extra aggregate industry prot generated from the change in the number of rms is equally divided
1
Subsection 8.4.2 can be presented using diagrams only. However, the rst printing of the rst edition
In
lacks a justication why rm 1s (high and low) best-response functions do not shift with a change in k.
the second printing I intend to add an appendix at the end of this chapter which derives both best-response
3 and not k
2 as wrongly
functions. Also, note that the equilibrium rst-period capital level in Figure 8.7 is k
stated in the rst printing.

30

Concentration, Mergers, and Entry Barriers


between the two unmerged groups of rms. Thus, this share of the change in
aggregate industry prot is lower than the prot each rm makes under the
triopoly.
In contrast, merging into a monopoly can only increase prot since a monopoly
can always produce like a duopoly, and under monopoly, all industry prot is
equally divided between the three rms.
3. (a) We look for a Nash equilibrium in prices.2 The X-seller takes pY as given and
solves
max X = pX Q = pX ( pX 2pY ),
pX

yielding pX =

pY .
2

The Y -seller takes pX as given and solves


max Y = pY 2Q = 2pY ( pX 2pY ),
pY

yielding pY =

pX
.
4

Hence,
pN
X =

2
N
= YN =
, pN
, pN
, QN = , X
.
Y =
S =
3
6
3
3
9

(b) We dene a system as one unit of X bundled with two units of Y . Then, the
merged rm chooses a system price pS that solves
max S = pS ( pS )
pS

yielding pS = QS =

2
, and S =
.
2
4

(c) To make a welfare judgment on the gains from this merger it is sucient to
compare prices and prot levels. Before the merger, the price of one system is
p0S = pX + 2pY = 2/3 > /2 = p1S which is the price after the merger. Also,
before the merger, aggregate prot is X + Y = 22 /9 < 2 /4 = S1 which is
the prot after the merger takes place. Since the system price falls and industry
prot increases, the merger is welfare improving.
4. In view of Denition 8.1, Figure 8.1 shows that the pair (pI , q I ) = (20, 10) constitutes
a contestable-market equilibrium.
Formally, the (incumbents) industry conguration (pI , q I ) = (20, 10) is feasible since
demand equals supply and the incumbent makes a nonnegative prot. It is also sustainable since no potential entrant can undercut the incumbents price while making
a positive prot.

A typo in the rst printing of the rst edition: the question should state that Q = x = y/2.

Concentration, Mergers, and Entry Barriers

pI

31

p = 60 4Q

AC(q I ) =
20

100
q

10

qI

Solution-Figure 8.1: Contestable-markets equilibrium

+q

Chapter 9
Research and Development
Most of the topics in this chapter are not technically demanding (and less tedious in terms
of technical derivations). The classications of process innovation (section 9.1) uses only
demand and supply diagrams. Innovation race (section 9.2) relies on simple discrete probability calculations. Cooperation in R&D (section 9.3) relies on a Cournot market structure
in the second stage and a Nash equilibrium in R&D in the rst stage. The calculation of the
optimal patent life (section 9.4) does not reach a closed-form solution. However Figure 9.3
enables the instructor to explain the cost and benet of the patent system without performing any calculations. Then, the instructor can simply reproduce the government-innovation
two stage game and demonstrate how to build the social welfare function for calculating the
optimal patent life. Licensing of innovation (section 9.5) uses supply and demand analysis.
The international analysis (section 9.6) is divided into a simple matrix game of product
innovation and a two stage subsidy Cournot game of process innovation. Finally, the appendix sections discuss patent law and R&D joint ventures (sections 9.7 and 9.8) and can
be assigned as homework reading.
Answers to Exercises
1. Using Denition 9.1, innovation is major if the monopoly price is below the initial
unit cost; that is if pM < c0 . Innovation is minor if pM > c0 . To nd pM , the
monopoly solves M R(QM ) = a 2QM = c1 = 2c0 a. Therefore, the monopolys
prot maximizing output is QM = a c0 . Hence, pM = c0 . Therefore, according to
Denition 9.1 the present innovation is neither major nor minor.
2. (a) The expected prot of a rm engaged in R&D, given that the other two rms are
also engaged in R&D, is the prize times the sum of the probability that the rm
discovers while the other two do not, plus twice the probability that it discovers
while one competing rm discovers and one does not, plus the probability that
all the three discover simultaneously, minus innovation cost. Formally, for each
rm i, i = 1, 2, 3,
111V
111V
7V
111
V +
(2) +
I =
I.
Ei =
222
222 2
222 3
24
Thus, all the three rms nd it protable to engage in R&D if V > 24I/7 = 24/7
(since I = 1).
(b) The now single rm can operate zero, one, or two labs.
Operating a single lab: In this case, E|1 lab = V /2 I.
Operating two labs: In this case, the probability of discovery is one minus the
probability that none of the labs discovers. Thus,


11
3V
2I.
E|2 labs = 1
V 2I =
22
4

34

Research and Development


Now, E|2 labs > E|1 lab if V > 4I, which constitutes a sucient condition
for having the rm choosing to operate two labs.
3. (a) The probability that a rm does not discover is (1 ). Hence, the probability
that none of the n rms discovers at a particular date is (1 )n .
(b) Clearly, one minus the probability that none discovers. Hence, 1 (1 )n .
(c) Similar to the derivation given in (9.5),
ET (n) = [1 (1 )n ] 1 + (1 )n [1 (1 )n ] 2
+ (1 )2 n [1 (1 )n ] 3 + (1 )3 n [1 (1 )n ] 4 +
Therefore,
n

ET (n) = [1 (1 ) ]


t=1

[(1 )n ]t1 t

(Lem 9.1)

1 (1 )n
1
.
=
[1 (1 )n ]2
(2 )

4. (a) If the EC provides Airbus with a subsidy of 10 (contingent on developing the


megacarrier, then Airbus dominant action is to develop.
(b) If the U.S. government provides Boeing with a subsidy of 10 (contingent on
developing the megacarrier, then Boeings dominant action is to develop. Note
that this U.S. subsidy is independent whether the EC provides Airbus with a
subsidy of 10 or 15.
(c) None! After the EC subsidy to Airbus is implemented, Airbus dominant action
is to develop (hence, independent of the U.S. subsidy to Boeing).
(d) Like any other welfare argument in economics, there are pros and cons to having
both governments subsidizing the development of the new megacarrier. Arguments against the subsidies include (i) duplication of R&D which require twice
the amount of resources needed to develop and test the plane, (ii) the government may not possess the means to determine whether the benets from the
development to their own economy dominate the cost, (i.e., economists generally
believe that the private sector can collect more accurate information), and (iii)
income distribution eects; that is, some economists claim that the development
of the Concord by some EC countries constitute a net transfer from poor to rich
people, since most poor and middle-class people do not get to y in the Concord.
Arguments for the subsidy include the introduction of more competition between
aircraft manufacturers that would reduce aircraft prices, boost the airline industry, and may therefore increase the amount of travel.

Chapter 10
The Economics of Compatibility and Standards
The introduction to this chapter provides some basic terminology and a simple Battle of the
Sexes game of product standardization, which can be taught to the less technically procient students. The behavior of a monopoly telephone company under network externalities
(Subsection 10.1.1) uses very simple calculus (basically one rst-order condition). The remainder of section 10.1 is devoted to a simple discrete (noncalculus) standardization-variety
tradeo model. The remaining two sections, the supporting services approach (section 10.2)
and the components approach (section 10.3) are somewhat more dicult despite the fact
that no calculus is used. In particular, whereas the components approach model is very
simple, some students may get confused because of the existence of multiple equilibria under
incompatibility, in which case, the instructor may want to construct only one equilibrium
and compare it to the equilibrium under compatible systems.
The important points to emphasize are:
A network externality is a particular type of a consumption externality, where a
consumers utility is aected by the number of consumers purchasing the same brand.
Thus, network externalities constitute an assumption about consumer preferences.
The supporting services and the components approaches do not assume any externality. The network eects, where the utility of a consumer is aected by the brand
choices of other consumers is an equilibrium result (rather than an assumption).1
Thus, the network eects are generated by having hardware and software (or other
types of hardware) treated as perfect complements.
Answers to Exercises
Unfortunately, the two exercises cover only the supporting services and the components
approaches. In future printings/editions, I intend to add two exercises covering the two
topics analyzed in the network externalities section.
1. (a) Firm A gains control over the entire market when = 1. In this case, by (10.10),
pB has to be suciently high to satisfy
1 = =
(b)

Y pA
,
2Y pA pB

or pB = Y.

i. Let denote the market share for rm A after income has doubled from Y
to 2Y . Then, by (10.10) we have that
Y pA p B
2Y pA pB
> =
=
4Y pA pB
2Y pA pB

Some authors use the term indirect network externalities to describe the behavior generated by the
supporting services and the components approaches. I feel that this term was poorly chosen since externalities
are either assumed or not assumed at all, and therefore the term indirect cannot be used to describe models
that do not explicitly assume any externality.

36

The Economics of Compatibility and Standards


if pA > pB . This result can also be demonstrated by dierentiating with
respect to Y , yielding
d
pA pB
>0
=
dY
(2Y pA pB )2

if pA > pB .

Thus, the rm with the lower market share increases its market share when
consumer income rises.
ii. Since increases, by (10.9) NA /NB must increase.
2. (a) Potential equilibria can be classied into two types: (i) Firm A sells to consumers
AA, and AB; and rm B sells to BB and BA (alternatively, Firm A sells to
consumers AA, and BA; and rm B sells to BB and AB). (ii) Firm A sells to
AA, AB, and BA; whereas rm B sells to BB only (alternatively, rm A sells
to AA only, whereas rm B sells to BB, AB, and BA).
In what follows we will demonstrate, by a way of contradiction, that both types
of equilibria do not exists.
i. Under this type, pIA = pIB since consumers AB and BA are indierent between systems AA and BB as long as the prices are equal. However, this
prices cannot support a Bertrand-Nash equilibrium since rm A can increase
its prot by undercutting rm B by charging pA = pIB + which will cause
consumer BA to switch to system AA (this is the usual Bertrand undercutting argument).
ii. Under this type, we must have that pIB = pIA + 2 (otherwise rm B is not
maximizing its prot). However, since consumers reservation utility is zero,
see equation (10.12), equilibrium prices must satisfy pIB 2, hence, pIA = 0
I = 0. However, rm A can always increase its prot by raising its
and A
price by + thereby making strictly positive prot.
(b) Following the proof of Proposition 10.14, it can be established that there exists
an equilibrium in which each consumer buys his or her ideal system. In this
equilibrium,
pxA = pyA = pxB = pyB = ,

c
c
and A
= B
= 4.

Chapter 11
Advertising
From a technical point of view, the analyses given in this chapter are pretty much straight
forward, perhaps, with the exception of the calculation of the socially optimal level of
persuasive advertising given in equation 11.13, where an integral is computed. The appendix
describing advertising regulations can be assigned as a home reading.
Answers to Exercises
1. (a) Since
+A =

%Q
= 0.05,
%A

and +p =

%Q
= 0.2,
%p

then by the Dorfman-Steiner condition (Proposition 11.1), we have that the prot
maximizing ratio of advertising to revenue is
AM
+A
1
=
= .
M
pQ
+p
4
Hence, AM = $2.5 mil..
(b) Now, +p = 0.5. Therefore AM = 10 0.05/0.5 = $1 mil..
(c) When the demand becomes more price elastic, the monopoly reduces the ratio
of advertising expenditure to revenue.
2. (a) A = A = 1 implies that n = n = 3, hence, = b = 30 1 = 29. When
A = A = 2, we have it that n = n = 3, hence, = b = 30 2 = 28.
Conclusion: if both rms spend the same amount on advertising, then industry
prot is maximized at A = A 0. That is, advertising in this city serves
as a mean by which each rm attracts consumers from the competing rm, but
cannot increase industry sales beyond the level determined by zero advertising.
(b) Fortune teller takes A as given and chooses A to maximize his or her prot.
The rst-order condition is given by
0=

30A
=
1.
A
(A )2

The
second-order condition can be easilyveried. Hence, A = R (A ) =
30A . By symmetry, A = R (A ) = 30A . The two best-response functions are drawn in Solution-Figure 11.1.
(c) Solution-Figure 11.1 illustrates that the two best response functions intersect
twice, reecting the fact that there exist two Nash equilibria for this advertising
game. For the purpose of this question, we dene 0/0 (yes, zero divided by
zero) as equal to 1. Then, clearly A = A = 0 constitutes one NE. To obtain

38

Advertising

R (A )

30 ..................................
.
...
...
..

..
..
..
..
..
..

..
..
..
.
0
30

R (A )

Solution-Figure 11.1: Fortune tellers advertising best-response functions




the other NE, substituting R (A ) into R yields A = 30 30A . Hence,


N
AN
= A = 30 which is also illustrated in Solution-Figure 11.1.
(d) In the interior NE, iN (30, 30) = 30 30 = 0, i = , , which is lower than
iN (1, 1) = 29. Thus, the industrys prot is not maximized at the interior NE.
However, at the other NE, industrys prot is maximized and each rm earns
iN (0, 0) = 30.
(e) The interior NE (30, 30) is stable. The interior NE (0, 0) is unstable since any unilateral deviation by one of the rm would trigger a sequential chain of responses
that would move the advertising equilibrium to (30, 30) (see Solution-Figure 11.1
for an illustration). Thus, in this example, the NE that maximizes industry prot
is unstable.
3. In order for (I, P ) to constitute a NE outcome, we need to show that unilateral
deviation is not protable for each rm. That is, using Table 11.1,
2 (I, P ) = N 1 (I, I) = (1 )E
1 (I, P ) = E 1 (P, P ) = N/2

when 1 N/E;
when N/(2E).

Chapter 12
Quality, Durability, and Warranties
This is a long chapter with a wide variety of topics dealing with supply and demand for
quality. The analyses are pretty much straight forward and do not require any preparation
with the exception of section 12.2 that relies on the knowledge of location models developed
in Chapter 7. Other sections use only simple calculations of expected values. The appendix
on the legal approach to liability (section 12.8) may be assigned as a home reading.
An Alternative Model for Section 12.6
The model suggested below provides some improvement as it assumes that consumers are
expected utility maximizers (similar to what is assumed in Section 12.5).1 Suppose that
consumers believe that the monopoly is a high-quality producer with probability , and a
low-quality producer with probability 1, where 0 < < 1. Then, without any additional
information, EU = H + (1 )L p. Therefore, the maximum price that consumers are
willing to pay is p = H + (1 )L. Then a low-quality producer could set the price at p,
and make a prot of
(
p cL ) 1 = H + (1 )L cL = H + (1 )L cL > 0.

()

The price and quantity pair in Proposition 12.10 should be modied to


pm = H

and q m =

H + (1 )L cL
.
H cL

Had the monopolist been a low-quality producer, he or she could have made the same prot
by setting p and selling to all consumers [as is indicated in (*)] rather than setting p = pm
and q = q m . Therefore, in choosing this price-quantity pair the monopolist could signal his
or her quality to the consumers. Finally, since H > L > cH > cL ,
(H cH )

H + (1 )L cL
> H + (1 )L cH = (
p cH ) 1.
H cL

Hence, the high-quality monopoly would indeed nd it protable to signal its high quality
to the consumers.

I thank Keiichi Koda for proposing this improvement.

40

Quality, Durability, and Warranties

Answers to Exercises
1. (a) Using the utility function (12.2) and the zero reservation utility assumption, the
consumer who is indierent between purchasing from A and not purchasing at
all must satisfy Uz (A) = az pA = 0, yielding z = pA /a.
(b) Follows directly from (12.4).
(c) For a given pB , rm A chooses pA to

max A = pA
pA

pB pA p A

,
ba
a

yielding 0 =

pB 2pA 2pA

.
ba
a

For a given pA , rm B chooses pB to

max B = pB 1
pB

pB pA
,
ba

yielding 0 = 1

2pB pA
.
ba

Second-order conditions are easily obtained. Solving the two rst-order conditions yield the equilibrium prices. Substituting into the prot functions yield the
equilibrium prot levels.
(d) We rst verify that given be = 1, the prot maximizing location of rm A is
ae = 4/7. Thus, given be = 1, rm A chooses ae to
max a =
a

a(1 a)
,
(4 a)2

4
yielding ae = .
7

We now verify that given ae = 4/7, rm B would choose to locate on the east
edge of the street.2 For a given ae rm B chooses be to maximize B which is
given above. We would like to show that for ae = 4/7,
B (4/7, b)
49b2 21b + 8 (b).
b
To demonstrate this inequality, we need to prove that (b) > 0 for all b [0, 1].
Clearly  (b) = 98b 21, which is negative for 0 b < 21/98 and positive
for b > 21/98. Hence, the function hits a minimum at b = 21/98. Now,
(21/98) = 5.75 > 0. Hence (b) = B /b > 0 for all b [0, 1] implying that
rm B would locate at be = 1.
0<

2. Let Ui (k) denote the utility of consumer i when he buys the brand with quality k,
k = H, L and i = 1, 2. We want to show that
U2 (L) = L(I2 pL ) > H(I2 pH ) = U2 (H).
From (12.1), we have it that since consumer 1 buys the low-quality brand, then
U1 (L) = L(I1 pL ) > H(I1 pH ) = U1 (H).
Therefore,

(H L)I1 < HpH LpL .

Now, since I2 < I1 by assumption, (H L)I2 < HpH LpL , implying that H(I2
pH ) < L(I2 pL ).
2

Many students will nd this part of the proof to be dicult.

Quality, Durability, and Warranties

41

3. Under these prices, we seek to characterize the demand and supply patterns of our
agents.3
New buyers (who do not yet own a car): Recalling our assumption that N L =
U L = 0, if a new buyer buys a new car, then his or her expected utility is V b =
0.5N G + 0.5N L pN = 0. In contrast, if a new buyer buys a used car then V b =
0.5U G + 0.5U L pU > 0. Hence, under the assumed prices, new buyers buy used cars.
Good-used-car sellers: The question simply assumes that good-used-car owners
must leave the country and therefore sell their good-used cars.
Lemon-used-car sellers: If a lemon-used-car seller sells his or her lemon and buys
a new car, then V s,L = 0.5N G pN + pU = pU . In contrast, if she or he buys a
used car then V s,L = 0.5U G pU + pU = 0.5U G . By assumption pU < 0.5U G , hence,
lemon-used-car sellers buy (and sell) used cars.
Altogether, under the assumed prices, nobody buys a new car.
4. (a) The monopolys production cost is the initial production cost plus the expected
(as most once) replacement cost, which is equal to c + (1 )c = (2 )c.
(b) The maximum monopoly price is the expected consumer surplus from purchasing
this product with this type of warranty. This surplus equals V if the product
does not fail at all and if the product fails only once (in which case it is replaced).
The probability of a double failure is (1)2 . Hence, the probability of no double
failure is 1 (1 )2 = (2 ). Therefore, pM = (2 )V .
(c) If the monopoly provides this warranty, then W = (2 )V (2 )c =
(2 )(V c). If the monopoly sells with no warranty, then N W = V c.
Hence, W > N W since it is assumed that V > c. Thus, the monopoly will
bundle the product with this warranty.

The purpose of this question is demonstrate that used cars are traded when some good-used-car owners
are forced to sell their car, say, because they leave the country. Note: If you are using the rst printing of
the rst edition, change the last sentence in this question to . . . the four types of agents. . . .

Chapter 13
Pricing Tactics: Two-Part Tari and Peak-Load Pricing
The analyses given in this relatively short chapter rely mainly on logical arguments and
therefore do not require any special technical preparation. Section 13.4 presents an attempt
to endogenize the seasons in determining prot maximizing peak-load pricing, and therefore
may be given a lower priority in case the instructor is short of time.1
Answers to Exercises
1. (a) The two cost functions are drawn in Solution-Figure 13.1.
$
T C(k) = 0.05k

T C(k) = 300 + 0.02k

300

k
(# copies)

Solution-Figure 13.1: The cost of photocopying with or without two-part tari


(b) The number of copies yielding identical cost under the two payment systems is
Therefore, k = 10, 000 copies. Thus, for
found by equating 0.05k = 300 + 0.02k.

the xed-per-unit
k > k, the two-part tari system is less costly, and for k < k,
price system is less costly.
2. (a) By Proposition 13.4, to nd the prot maximizing pricing structure we need (i)
to equate the high-season marginal revenue to the cost of an aircraft seat plus
the per-passenger cost, and (ii) to equate the low-season marginal revenue to
the per-passenger cost only.
Given that the Summer demand curve lies completely below the Winter demand
curve, we can conclude that Winter is the high season. However, since the two
demand curves are relatively close, we need to be careful about the capacity
constraint determined by the Winter condition.
In the Winter, capacity is determined by having the monopoly equating
M RW (qW ) = 10 2qW = r + c = 2,

and M RS (qS ) = 5 qS = c = 1.

1
If you are using the rst printing of the rst edition note that the calculations of consumer surplus in
section 13.2 are wrong. Ask for the errata le by e-mail.

44

Pricing Tactics: Two-Part Tari and Peak-Load Pricing


The rst equality implies that qW = 4, hence, pW = 6. The second equality
implies that qS = 4, hence, pS = 3. Altogether, I = 64+342414 = 24.
(b) Similar to the previous subquestion, the monopoly equates
M RW (qW ) = 10 2qW = r + c = 4,

and M RS (qS ) = 5 qS = c = 1.

The rst equality implies that, qW = 3, hence, pW = 7. The second equality


implies qS = 4 > 3 = qW . Hence, due to the capacity constraint, the airline
will y only 3 passengers during the Summer season. Thus, pS = 5 3/2 = 3.5.
Altogether, I = 7 3 + 3.5 3 4 3 1 3 = 16.5.

Chapter 14
Marketing Tactics: Bundling, Upgrading, and Dealerships
This chapter introduces several topics related to prot enhancing marketing tools that were
not discussed (at least not in full) in earlier chapters.
Answers to Exercises
1. (a) Under pure tying, we need to check two options: (i) Under pT = 4, all three
consumers buy the packages, hence, T (4) = 3 4 = 12. (ii) Under pT = 10,
only consumer 2 buys a package, hence, T (10) = 10. Clearly, the rst option
maximizes the prot under pure tying.
T = pM T = 4 and
(b) The prot maximizing product and package prices are pM
X
Y
M
T
p
= 8. Under this price structure, consumer 1 buys one unit of X, consumer 3
buys one unit of Y and consumer 2 buys the package (or one unit of each good).
Altogether, M T = 4 + 4 + 8 = 16.

(c) Notice that mixed tying yields the same prot maximizing pricing structure as
no tying, since in the present example, the monopoly cannot charge consumer 2
more than the sum of the prices of the two products sold separately. In fact,
in the present example, pure tying is prot reducing. Therefore, mixed tying
(which yields an identical allocation as no tying) generates a higher prot than
pure tying.
2. (a) We need to compare the prot levels under the commitment of not introducing
a new edition, in which case the publisher can charge a higher price in the
rst period by adding the resale value to the price, to the prot level when no
commitment is made and the monopoly introduces a new edition.
By (14.17), the prot level under commitment is nV , and under no commitment
is 2n(V c) F . Comparing the two prot levels reveals that commitment not
to introduce a new edition is more protable if F > nV 2c.
(b) Indeed, this condition in weaker (i.e., implied by) the condition given in (14.17)
since it compares in the case of commitment, in which rst-period students believe that no new edition will be published, hence the monopoly can raise the
books price by its resale value, to the case where there is no commitment and a
new edition is introduced.
In contrast, the condition given in (14.16) and hence in (14.17) is derived by
comparing the monopolys second period prot only and therefore yields the
condition under which a new edition will not be introduced in the second period. In other words, the condition given in (14.16) and (14.17) does not take
into account that rst period price can be raised by the resale value when a
commitment is made not to introduce a new edition.

46

Marketing Tactics: Bundling, Upgrading, and Dealerships


3. (a) When a new edition is not introduced in the second period, the monopoly competes with period 1 students on selling the rst edition. First period students will
undercut the monopoly (since rst-period students do not have any production
U
cost) and for a given pN
2 will set p2 so that
U
V pN
2 V p2 .

Now, in a Bertrand equilibrium, if used-book sellers can undercut the monopolist,


then the monopoly price is reduced to cost( pN
2 = c). Hence, the maximum price
user-book sellers can charge is


pU
2

c (1 )V
0

if c (1 )V
otherwise

Notice that if c < (1 )V , or < 1 c/V , then the monopoly undercuts the
used-book sellers (since consumers do not value used books very much). Under
this condition, the monopoly would set pN
2 = (1 )V , and therefore earn a
N
prot of 2 = n[(1 )V c].
(b) There are two cases:
High value for used books ( > 1 c/V ): A new edition would yield a
second-period prot of 2N = n(V c)F . When a new edition is not introduced,
then 2 = 0. Hence, a new edition is introduced if F n(V c), which is the
same as condition (14.16) when = 1.
Low value for used book ( < 1 c/V ): When is low, used book sellers
are undercut by the monopoly. In this case, if a new edition is not introduced in
the second period, then pN
2 = (1 )V c (the monopoly undercuts used-book
sellers). Hence, 2N = n[(1 )V c]. Altogether, a new edition is introduced
if n(V c) F n[(1 )V c], or if F nN , which is dierent from
condition (14.16).
4. (a) Under this contract, the dealer chooses Q (sales) to
max d = (1 )[p(Q)Q cQ] = (1 )[(a Q)Q cQ].
Q

The rst-order condition yields Qd = (a c)/2, pd = (a + c)/2, and d =


(1 )(a c)2 /4. Thus, the dealers sales and price are independent of (1 )
(share of the prot of the dealer). Under this contract, the industry prot is
M + d =

(a c)2
(a c)2
(a c)2
+ (1 )
=
,
4
4
4

which equals the industrys prot when the rms vertically integrate into a
monopoly given in (14.24).

Marketing Tactics: Bundling, Upgrading, and Dealerships

47

(b) Under this contract, the dealer chooses Q that solves


max d = (1 )p(Q)Q cQ = (1 )(a Q)Q cQ.
Q

The rst-order condition is (1 )(a 2Q) c = 0, hence,


c
c
a
a
.
, and pd = +
Qd =
2 2(1 )
2 2(1 )
Now, industry prot is given by the sum
M + d =

Qd pd + (1 )Qd pd cQd = pd c Qd

a c 2(1 )c
a
c
=
+

2
2(1 )
2 2(1 )



ac
a
c
=

2
2 2(1 )
2
(a c)
<
.
4

Thus, this contract yields a lower prot than the industrys prot when the rms
vertically integrate into a monopoly given in (14.24).
(c) Whereas the prot sharing contract analyzed in part (a) is optimal (i.e., maximizes industrys prot) it is rarely observed since it requires having the manufacturer monitoring the prot made by the dealer. This monitoring may be too
costly for the manufacturer. In contrast, the per-unit contract analyzed in the
text, despite being not optimal, does not require any monitoring since the dealer
pays up front for each unit it acquires from the manufacturer.
5. We will demonstrate how a monopoly can benet from establishing a trade-in procedure by constructing a simple model. Suppose that there are two consumers: one
owns an old refrigerator and one that is a new buyer. The utility from using an old
refrigerator is VU whereas the utility from buying a new one is max{VN pN , 0},
where VN > VU > 0. We further assume that VN < 2VU .
Single price policy: I will argue that the prot maximizing price is pN = VN ,
thereby selling only to the rst-time buyer and earning a prot of = VN . To see
this, note that the monopoly can set a lower price, pL = VN VU , thereby selling two
units and earn a prot of L = 2(VN VU ). However, by assumption, L < .
Trade-in price policy: We now show that by employing a trade-in price policy, the
monopoly can price discriminate between new buyers and old-model owners. Suppose
that the monopoly announces that current users can trade-in their old refrigerator for
a new one for a price of pT I = VN VU , whereas the price with no trade-in is pN = VN .
Note that under these prices, current users trade-in their old refrigerator for a new
one, and also a new buyer buys a new one. Altogether, T I = VN VU + VN =
2VN VU > VN = .

Chapter 15
Management, Compensation, and Regulation
Instructors who choose to teach from this chapter are urged to go over the principal-agent
problem (section 15.1), which is developed three times in order to allow for a a gradual increase in the degree of diculty. More precisely, the rst subsection illustrates the problem
with no uncertainty. A subsequent subsection introduces uncertainty with no risk aversion.
The third subsection of introduces risk aversion implicitly by assuming asymmetric information. Altogether, these three subsections analyze the principal-agent problem in dierent
degrees of complexity, thereby enabling the instructor to stop at the level that exceeds the
students ability.
Section 15.2 introduces a simple free-rider team eort problem. Section 15.3 introduces
Fershtman-Judd model, which is recommended only if the students are capable of solving
Cournot-type problems very easily. The remaining sections: executives compensation (section 15.4) and the regulation of the rm (section 15.4) do t the more advanced students.
Answers to Exercises
1. (a) Under collusion, the social planner chooses a common eort level for all scientists e that solves


max w e2 =
e

V
Ne
e2 =
= e e2 ,
N
N

yielding e = 1/2.
(b) Each scientist i takes all other eort levels (ej , j = i) as given and solves
V
(ei )2 =
max Ui =
ei
N

j=i ej

+ ei

(ei )2 ,

yielding a NE where ei = 1/(2N ) for all i = 1, 2 . . . N .


(c) Yes, since the optimal eort level is e = 1/2 which is independent of the number
of scientists. In constrast, the NE eort levels decrease further below the optimal
level when the number of scientists increases, reecting a stronger free-rider eect.
2. (a) The manager of rm 1 chooses q1 that solves
max M1 = 0.5[(a q1 q2 )q1 + (1 c)q1 ],
q1

yielding a best-response function given by


q1 = BR1 (q2 ) =

a c + 1 q2
.
2

50

Management, Compensation, and Regulation


The best response function of rm 2 (the rm that maximizes prot only) is
given by q2 = BR2 (q1 ) = 0.5(a c q1 ). Therefore, the equilibrium output
levels are given by
q1e =

ac+2
,
3

and q2e =

ac1
.
3

Clearly, q1e > q2e conrming that the managerial compensating scheme of rm 1 is
output increasing at the expense of the rm that maximizes prot only (rm 2).
(b) The owner of rm 1 earns 1O = 1 M1 . From the previous calculation we
conclude that pe = (a 2c 1)/3. Hence, 1e = (a c 1)(a c + 2)/9. We
now set 1 suciently small (as in [15.29]), so that we approximate the owners
prot by the rms prot. Formally, 1O = 1 .
Finally, comparing the prot of the owner of rm 1 under the present managerial
compensation scheme to her prot when this manager maximizes prot only (the
simple Cournot prot given in (6.7)) yields that 1O 1c if and only if a 2 + c.
That is, the present compensation scheme dominates a simply Cournot behavior
(given that the other rm behaves in a Cournot fashion) if the demand intercept
is suciently higher than the unit production cost.

Chapter 16
Price Dispersion and Search Theory
Price dispersion (section 16.1) demonstrates that a discount store and an expensive store
can coexist in a market where consumers have dierent search costs (resulting, say, from
dierent values of time). The analysis relies on a simple of location model. Thus, the
students should nd this section easy to follow. Search theory (section 16.2) is introduced
with no calculus, so the analysis relies on simple probability arguments.
Answers to Exercises
1. (a) To see what happens, we look at the equilibrium prices when L 0. From (16.9)
we see that
lim peD = lim peN D = 2H and se 0.
L

Hence, when L becomes small (i.e., some consumers have a negligible search cost),
both stores charge the same price, implying that there is no discount store.
(b) When = 3/2, the consumer indierent between searching or buying at random
is located at L. Hence, when = 3/2 all consumers buy at random. When
= 1, se < L implying that the discount store a negative market share (which
is ruled out by out assumption that > 3/2).
2. (a) Using (16.13), we have it that
s
p

0
1

1
4.77

2
6.52

3
7.86

4
9.00

5
10.0

Solution-Table 16.1: Reservation price as a function of the search cost parameter


(b) Solution-table 16.1 reveals that for s 4 the consumer reservation price is p = 9,
implying that the consumer will accept any price in the rst store visit.
(c) Solution-table 16.1 reveals that p = 1 when s = 0 (i.e., when search cost is zero).
(d) When s = 0,1 the probability that the consumer nds the price higher than 1
(i.e., the reservation price) in a single store visit is = 8/9. The probability that
prices exceed 1 in two store visits is 2 = (8/9)2 . Hence, the probability that
this consumer nds prices exceed 1 in all store visits is limT (8/9)T = 0.
Note also that this result can be inferred from (16.16) where the calculated
expected number of stores visited is = 1/(1 ) = 9 < +.

1
The rst printing of the rst edition does not specify which search cost is assumed. This question should
read, under the search cost you found in the previous subquestion, calculate....

Chapter 17
Miscellaneous Industries
The purpose of this chapter is to demonstrate to the student that there is no general model
that can describe all industries; however, I also wish to demonstrate in this last chapter
that the tools developed in this book are indispensable to the analysis of most industries,
and are, therefore, worth learning.
This chapter discusses four types of industries that require special attention since (perhaps like any other industry) they do not fall in any category and therefore cannot be
analyzed by simply picking one of the market structure analyzed earlier in the book. In
future editions I will attempt to expand this part of the book by introducing at least three
more industries: the health industry, the defense industry, and (you guessed, of course), the
lawyers.
Answers to Exercises
1. From the denition, this technology (cost function) exhibits economies of scope if
T C(n, n, n) = (n + n + n )
< (n ) + (n ) + (n ) = T C(n, 0, 0) + T C(0, n, 0) + T C(0, 0, n),
or,

(3n ) < 3n ,

therefore if 3 < 3,

hence if < 1.

2. The main message here is that the introduction of a binding price cap by the regulator
will cause the monopoly airline to reduce ight frequency (or any other quality aspect
of the service).
(a) Under the FC network, each route can be analyzed separately. Now, from the
utility function of route 3 passengers
(17.2), we see that the monopoly extracts

maximum surplus when p3 = + f3 . Since the airfare is regulated (i.e., p3 = p3 )


we have it that the minimum frequency that the airline must provide on route 3
(at this airfare) is f3 = (
p3 )2 . Clearly, as p3 decreases, the corresponding

frequency f3 decreases.
(b) Under the HS network, the ight frequency on route 3 is the (minimum) frequency on routes 1 and 2.1 Hence, similar to (17.5), but with a given p3 , the
monopoly chooses f f1 = f2 that solves


p3 + 2n + 2n f 2cf,
max h = n
f

p3 .

The rst printing of the rst edition contains a typo: Replace ...charges an airfare f3 with airfare

54

Miscellaneous Industries


yielding a rst order condition given by 0 = 2n/(2 f) 2c. Hence,2


f =

n
2c

<

3n
4c

= f h.

Therefore, under the HS network, a price cap on route 3 will reduce the ight
frequency on all routes.
3. The analysis here follows the same stages as in section 17.4, but it uses a dierent
driving time function.3
(a) If some people take the train and some drive a car, it must be that LT = LC .
Thus,


v + 1/3
3
e
.
v + = v(nC ) , implying that nC =
v
(b) The social planner chooses nsC that solves
min Ls = (N nc )(v + ) + nC V (nC )3 ,
yielding
nsC =

v+
4v

1/3

< neC .

Thus, the social planner would recommend a policy that would result in a reduction in the number of people who drive a car.
(c) Let denote the highway toll on this route. Then, to set the toll that would
generate the socially optimal number of highway users, the social planner sets s
that solves


v+
s 3
+ ,
v + = v(nC ) + = v
4v
yielding that s = 3(v + )/4.

2
One has also to check that, in this equilibrium, the airfare on route 3, p3 , is lower than the sum of the
airfares on routes 1 and 2; that is, we need to verify that (or nd the conditions under which) p3 p1 + p2 .
Otherwise, route 3 passengers can purchase two separate tickets and reach their destination at a lower airfare.
3
The rst printing of the rst edition contains a typo. Simply set tC = (nC )3 (the equation stated in this
question equals to the value of the driving time, LC and not tC ).

You might also like